LSAT and Law School Admissions Forum

Get expert LSAT preparation and law school admissions advice from PowerScore Test Preparation.

 Administrator
PowerScore Staff
  • PowerScore Staff
  • Posts: 8916
  • Joined: Feb 02, 2011
|
#40016
Complete Question Explanation
(The complete setup for this game can be found here: lsat/viewtopic.php?t=12912)

The correct answer choice is (D)

If H and L are assigned to the same ceremony as each other, then M cannot be assigned in that ceremony, because the second rule forbids L and M from being in the same group. Notice, however, that if K is not assigned to Thorne, then H and M would have to be assigned to the same ceremony as each other in compliance with the fourth rule. Since the necessary condition cannot be satisfied here, we must ensure that K is assigned to Thorne.

This inference can also be validated by referencing Templates 2A and 2B in the setup explanation (in Template 1, H and L are assigned to different ceremonies).

Answer choice (A) is incorrect, because F does not have to be assigned to Thorne. See Template 2A.

Answer choice (B) is incorrect, because G can also be assigned to Thorne, or else be unassigned. See Template 2A.

Answer choice (C) is incorrect, because G could be assigned to either ceremony without violating any of the conditions. See Templates 2A and 2B.

Answer choice (D) is the correct answer choice, because assigning K to Thorne is the only way to avoid triggering the sufficient condition in the fourth rule.

Answer choice (E) is incorrect, because L could also be assigned to Silva, or else be unassigned. See Template 2A.

Get the most out of your LSAT Prep Plus subscription.

Analyze and track your performance with our Testing and Analytics Package.